suppose that R(x) is a polynomial of degree 12 whose coefficients are real numbers. Also, suppose that R(x) has the following zeros. Answer the following.Edit: if its ok please double-check the answers.

Suppose That R(x) Is A Polynomial Of Degree 12 Whose Coefficients Are Real Numbers. Also, Suppose That

Answers

Answer 1

According to the polynomial roots description, we can conclude that 5-i is another zero of R(x), the maximum number of real zeroes that R(x) can have is 8, and the maximum number of non-real zeroes that R(x) can have is 4.

It is given to us that -

R(x) is a polynomial of degree 12 whose coefficients are real numbers.

The zeroes of R(x) are -

-6, -2+3i, -2-3i, 5+i

We have to find out -

(a) Another zero of R(x)

(b) The maximum number of real zeroes that R(x) can have

(c) The maximum number of non-real zeroes that R(x) can have

Since R(x) is a polynomial of degree 12, there are in total 12 roots.

(a) If a non-real, complex, root 5+i exists, then so it's conjugate 5-i exists which is another zero of R(x).

(b) Now, the total number of zeroes of R(x) can be stated as -

-6, -2+3i, -2-3i, 5+i, 5-i

From the above list of zeroes, we can see that there are at least 4 non-real zeroes. So, R(x) has a total of 12 zeroes and we know that there are at least 4 non-real zeroes.

Thus, we can say that the maximum number of real zeroes that R(x) can have is 12-4 = 8 real zeroes.

(c) The non-real complex zeroes always exist in conjugate pairs. And the total number of zeroes that R(x) can have is 12. And we already have determined that R(x) can have a maximum of 8 real zeroes.

Thus, the maximum number of non-real zeroes that R(x) can have is 12-8 = 4 non-real zeroes.

Therefore, according to the polynomial roots description, we can conclude that 5-i is another zero of R(x), the maximum number of real zeroes that R(x) can have is 8, and the maximum number of non-real zeroes that R(x) can have is 4.

To learn more about polynomial roots visit https://brainly.com/question/11906453

#SPJ9


Related Questions

Please help me I don’t know if I’m right or missing any other to select.

Answers

The given equations are

[tex]-x+4y=7[/tex][tex]6x-3y=42[/tex]

To find the answer we need to cancel out x or y.

so we have to find the LCM of the coefficients of the corresponding variable.

consider the coefficients of x is -1 in the first equation and 6 in the second equation .

Lcm of -1 and 6 is 6.

Multiplying the first equation by 6.

consider the coefficients of y is 4 in the first equation and -3 in the second equation .

Lcm of 4 and 3 is 12

Multiplying the first equation by 3 and the second equation by 4.

Either one of these is the first step to eliminate variables.

Amswer os

Please helpIf the 100th term of an arithmetic sequence is 595, and its common difference is 6, thenits first term a1= ,its second term a2= ,its third term a3=

Answers

Given

100th term of an arithmetic sequence is 595 and common difference , d = 6

Find

First three terms of arithmetic sequences.

Explanation

As we know the general nth term of an arithmetic sequence is given by

[tex]a_n=a+(n-1)d[/tex]

we have given 100th term = 595 , so

[tex]\begin{gathered} a_{100}=a+(100-1)6 \\ 595=a+99\times6 \\ 595-594=a \\ a=1 \end{gathered}[/tex]

so , first term = 1

second term = a + 6 = 7

third term = a + 2d = 1 +2*6 = 13

Final Answer

Therefore , the first terms of an arithmetic sequences are

[tex]a_1=1,a_2=7,a_3=13[/tex]

Additional and SubtractionAnswers should have only three significant figures.Question a) 86.5-0.07 ?Question b) 30.61-87.3-42.109 ?

Answers

a. 86.5-0.07

The result of the subtraction is:

[tex]\begin{gathered} 86.50 \\ -0.07 \\ ----- \\ 86.43 \end{gathered}[/tex]

As the answer should have only three significant figures then, the result is: 86.4

b. 30.61-87.3-42.109

The subtraction is:

[tex]\begin{gathered} 30.61 \\ -87.30 \\ ----- \\ -56.690 \\ -42.109 \\ ------ \\ -98.799 \end{gathered}[/tex]

As the answer should have only 3 significant figures, then the result is -98.8

The graph shows the number of gallons of white paint that were mixed with gallons of blue paint in various diffrent ratios:

Answers

From the graph, we can note that the points are in a line.

Hence, we must find the line equation for these points.

The general form of the straigh line equation is

[tex]y=mx+b[/tex]

where m is the slope and b the y-intercept. The slope can be computed as

[tex]m=\frac{y_2-y_1}{x_2-x_1}[/tex]

where,

[tex]\begin{gathered} (x_1,y_1)=(2,4) \\ (x_2,y_2)=(6,12) \end{gathered}[/tex]

By substituying these values into m, we have

[tex]m=\frac{12-4}{6-2}[/tex]

hence,

[tex]\begin{gathered} m=\frac{8}{4} \\ m=2 \end{gathered}[/tex]

the form of the line equation is

[tex]y=2x+b[/tex]

where x is the blue paint and y the white paint.

In order to find b, we can substitute one point into the above equation. For instance, the point

(2,4):

[tex]\begin{gathered} 4=2(2)+b \\ 4=4+b \\ b=0 \end{gathered}[/tex]

Thefore, the line equation is

[tex]y=2x[/tex]

Hence, the number of galons when we mix 1 gallon of blue pain is

[tex]\begin{gathered} y=2(1) \\ y=2 \end{gathered}[/tex]

in other words, for 1 gallon of blue paint we must have 2 gallons of white paint

place the letter of the angle relationship that beat represents the given angle pair in the box.

Answers

Angle relationship that best represents the given angle

Considering the angles in the attached diagram above

<6 and <13 has no relationship because they are not parallel

Hence NO relationship between <6 and <13

Determine the measures of the unknown angle.

Answers

To find the measure of the unknown angle we can use the triangle sum theorem that states that the sum of the measures of the interior angle of a triangle is 180°. We know the measure of two of the interior angles of the triangle that are 50° and 88°, and we can use this information to find the unknown one:

[tex]\begin{gathered} 50+88+\measuredangle3=180 \\ 138+\measuredangle3=180 \\ \measuredangle3=180-138 \\ \measuredangle3=42 \end{gathered}[/tex]

The correct answer is C. 42°.

13. Rose's probability of successfully shooting a basketball is 2/5. What is the probability of hershooting in at least 1 if she makes 4 shots?

Answers

Answer:

0.8704

Explanation:

The probability of successfully shooting a basketball is 2/5, so the probability t3/o fail will be equal to:

[tex]P=1-\frac{2}{5}=\frac{3}{5}[/tex]

Then, we will calculate the probability of failing the 4 shots. So, we need to multiply 3/5 by itself 4 times.

[tex]P(\text{fail all times) = }\frac{3}{5}\times\frac{3}{5}\times\frac{3}{5}\times\frac{3}{5}=0.1296[/tex]

Finally, the probability of her shooting in at least 1 if the complement of the probability to fail all times, so, the answer is:

[tex]P(at\text{ least 1) = 1 - 0.1296 = 0.8704}[/tex]

Therefore, the answer is 0.8704

Use your compass to help with the direction. Also, the question is in the question box

Answers

1. Extending the dashed lines

2. Translating the triangle ABC in the direction EF

copy the vector in each vertice

then with the final points draw the new triangle a distance of EF

The blue triangle is the translated triangle (in your case you can your compass to help with the direction and protractor to verify the distance).

Express: 12x-9x-4x+3 in factored form

Answers

SOLUTION:

Step 1:

In this question, we are given the following:

Expressing:

[tex]12\text{ x - 9x - 4x + 3}[/tex]

Step 2:

The details of the solution are as follows:

[tex]\begin{gathered} 12\text{ x -9x - 4 x + 3} \\ \text{= -x + 3} \\ =\text{ -\lparen x -3\rparen } \end{gathered}[/tex]

CONCLUSION:

The final answer in factored form =

[tex]-(x-3)[/tex]

v+8 over v = 1 over 2

Answers

The given expression is

[tex]\frac{v+8}{v}=\frac{1}{2}[/tex]

First, we multiply 2v on each side.

[tex]\begin{gathered} 2v\cdot\frac{v+8}{v}=2v\cdot\frac{1}{2} \\ 2v+16=v \end{gathered}[/tex]

Then, we subtract v on each side.

[tex]\begin{gathered} 2v-v+16=v-v \\ v+16=0 \end{gathered}[/tex]

At last, we subtract 16 on each side.

[tex]\begin{gathered} v+16-16=-16 \\ v=-16 \end{gathered}[/tex]Therefore, the solution is -16.

Question 4 5 points)Part 1: Find the median of the Science Midterm Exam Scores (2 points)Part 2: Explain how you found the median of the Science Midterm Exam Scores. Be sure to explain the process you used to identity at themedian is. (3 points)

Answers

median = 75

See explanation below

Explanation:

Part 1:

To find the emadian, we can state the data on the dot plot of the science midterm scores:

60, 65, 65, 70, 70, 75, 75, 75, 80, 80, 85, 85, 90, 95, 100

Total number of data set = 15

median = (N+1)/2

N = 15

Median = (15+1)/2 =16/2 = 8

Median = 8th position in the data

The 8th number = 75

Hence, the median of the science midterm scores = 75

Part 2:

The process is to write out all the data on the plot.

Count the number of data.

Then apply the median formula

Or because it is an odd number, the middle number after listing it out is the median.

The middle number here is 75

if triangle ABC has sides of length 9, 15, and 3x, between which two numbers must the value of x lie?

Answers

Let's employ the triangle inequality here.

If the sides were to form a triangle.

Then if 3x was the longest side, it must be less than the sum of 15 and 9, being the other 2 sides.

So;

[tex]\begin{gathered} 3x<15+9 \\ 3x<24 \\ x<8 \end{gathered}[/tex]

If 3x was the shortest side, then 15 would be the longest side, and thus

3x plus 9 must be greater than 15,

So;

[tex]\begin{gathered} 3x+9>15 \\ 3x>15-9 \\ 3x>6 \\ x>2 \end{gathered}[/tex]

So, the range of values for which x must lie is;

[tex]2i.e any values greater than 2 but less than 8.

If you bought 12 gallons of gas for $26.00, how much did you pay per gallon?

Answers

To get pay per gallon, we divide the total payment by the total amount of gallons.

So,

Total Cost = 26

Total Gallons = 24

Pay Per Gallon = 26/24 = $1.08 per gallon

LEVEL B 1.b) Solve for x angle relationship X+34" 2x-120

Answers

Answer

x = 46 degrees

Step-by-step explanation:

Alternate interior angles are equal

x + 34 = 2x - 12

Collect the like terms

x - 2x = -12 - 34

-x = -46

Divide both sides by -1

-x/-1 = -46/-1

x = 46 degrees

Hence, the value of x is 46 degrees

Imagine you are working for Hasbro making Gummy Bear containers. On a day to day basis you fill up two different size containers with gummy bears. One of the containers is4.4x5.7 x 6.0 in dimensions and contains 385 gummy bears. The other is 8.1 x 8.1 x 8.3 in dimensions. About how many gummy bears would fit in the box? Round to the nearestwhole number

Answers

It is given that,

One of the containers is 4.4 x 5.7 x 6.0 in dimensions and contains 385 gummy bears.

So, 1 gummy bear occupies,

[tex]\frac{4.4\times5.7\times6.0}{385}=0.39086[/tex]

The other is 8.1 x 8.1 x 8.3 in dimensions.

So, the number of gummy bears would fit in the box is,

[tex]\frac{8.1\times8.1\times8.3}{0.39086}=1393.24[/tex]

Hence, the number of gummy bears is 1,393 (Rounded to the nearest whole number).

y = 2x – 2 y = -x + 7

Answers

Given the system of equations:

[tex]\begin{gathered} y=2x-2 \\ y=-x+7 \end{gathered}[/tex]

We will find the solution of the system by the graph

To draw each line, we need to know 2 points

So, we will substitute with 2 values of x and calculate the corresponding value of y

For the first equation: y = 2x - 2

[tex]\begin{gathered} x=0\rightarrow y=2\cdot0-2=-2 \\ x=2\rightarrow y\rightarrow=2\cdot2-2=2 \end{gathered}[/tex]

So, the line passes through the points ( 0, -2 ) and ( 2, 2)

For the second line: y = -x + 7

[tex]\begin{gathered} x=0\rightarrow y=0+7=7 \\ x=2\rightarrow y=-2+7=5 \end{gathered}[/tex]

so, the line passes through the points ( 0, 7) and ( 2, 5)

The graph of the system will be as shown in the following figure:

As shown in the figure:

Equation 1 is the blue line

Equation 2 is the red line

The point of intersection = ( 3, 4)

So, the answer is the solution of the system = ( 3, 4 )

I need help with this please, I know that the opposite of 4.6 is -4.6 but I don’t know how to explain it.

Answers

Answer:

As a rational number is a fraction we had to convert our number to a fraction, and as the opposite number is the number with the same magnitude but a different sign, we had to change the sign.

[tex]-4\cdot\frac{3}{5}[/tex]

Explanation

• Rational numbers are the numbers that can be written as the fraction of two integers.

,

• Additionally, opposite numbers are numbers with the same magnitude but different signs.

Thus, based on these definitions, we have to change the sign and search for a fraction.

Steps:

0. From 4.6 we go to -4.6.

,

1. We convert -4.6 to a fraction: -4 is the whole number and we are left with -0.6, which is 6/10 (as it is in the tenth's place).

,

2. Simplifying 6/10 to 3/5 dividing both numbers by 2.

Heart Rates For a certain group of individuals, the average heart rate is 74 beats per minute. Assume the variable is normally distributed and the standarddeviation is 2 beats per minute. If a subject is selected at random, find the probability that the person has the following heart rate. Use a graphing calculator.Round the answers to four decimal places.Higher than 73 beats per minute,P (x> 73) =

Answers

we need to determine P (x> 73)

when

mean: μ = 74 beats/min

standard deviation: σ = 2 beats/min

First we need to use the following formula:

[tex]z=\frac{x-\mu}{\sigma}[/tex]

where

x = 73

μ = 74

σ = 2

and

Z is the z-score

... therefore

[tex]z=\frac{73-74}{2}=-\frac{1}{2}=-0.5[/tex]

If we check a table of z scores, we will find that when z = -0.5, then P = 0.3085

Now, since we need P(x>73)

therefore

[tex]P=1-0.3085=0.6915[/tex]

P(x>73) = 0.6915

Mason calculated the sales tax on his clothing purchase to be $5.57375. Round to the nearest hundredth. ANS $ __________

Answers

As per given by the question,

There are given that the sales tax is $5.57375.

Now,

For find the value nearest hundredth,

Nearest hundredth is the second digit after the decimal point.

That means,

If there is given that the value , x.yzw

Then, the nearest hundredth number is x.yz.

So,

From the given value.

The nearest hundredth value is, secon digit after the decima.

Here, second digit after the decimal is 57.

Then,

The nearest value of hundredth is 5.57.

Hence, the value is $5.57

I am very confused can you help me please thanks!

Answers

Solution

For this case we know that :

1/8 of teaspoon for every 3 cups of frosting

Now the amount of cups increase to 4 cups then we can find the number teaspoon

We can use a proportional rule and we got:

[tex]\frac{\frac{1}{8}}{3}=\frac{x}{4}[/tex]

The answer is:

C

Some of the tallest crystals in a cave in Mexico are 85 feet tall. Lin is 6 feet tall. About how many times as tall as Lin are the tallest​ crystals?


What is 85 / 6
/= divided by

Answers

14.16 ≈ 14 times as tall as Lin are the tallest​ crystals.

Given:

Some of the tallest crystals in a cave in Mexico are 85 feet tall.

Lin is 6 feet tall.

Number of times = tallest crystals height / lin height.

= 85 feet / 6 feet

= 14. 16 ≈ 14 times

14.16 is not represented in times so we take the nearest number which is 14.

Therefore 14 times as tall as Lin are the tallest​ crystals.

Learn more about the times and tallest crystals here:

https://brainly.com/question/12327373

#SPJ1

Lizzy is tiling a kitchen floor for the first time. She had a tough time at first and placed only 6 tiles the firstday. She started to go faster and by the end of day 4, she had placed 36 tiles. She worked at a steady rateafter the first day. Use an equation in point-slope form to determine how many days Lizzy took to placeall of the 100 tiles needed to finish the floor. Solve the problem using an equation in point-slope form.

Answers

We know that

• She placed 6 tiles on the first day.

,

• By the end of day 4, she had placed 36 tiles.

Based on the given information, we can express the following equation.

[tex]y=3x+6[/tex]

If she had placed 36 tiles in 3 days, it means she had placed 12 tiles per day, that's why the coefficient of x is 3. And the number 6 is the initial condition of the problem, that is, on day 0 she placed 6 tiles.

Now, for 100 tiles, we have to solve the equation when y = 100.

[tex]\begin{gathered} 100=3x+6 \\ 100-6=3x \\ 3x=94 \\ x=\frac{94}{3} \\ x=31.33333\ldots \end{gathered}[/tex]Therefore, she needs 32 days to place all the tiles.

Notice that we cannot say 31 days, because it would be incomplete.

raw the hyperbola for each equation in problem l. the partial

Answers

B. given the equation of the hyperbola :

[tex]\begin{gathered} 9x^2-y^2=9 \\ \\ \frac{x^2}{1}-\frac{y^2}{9}=1 \end{gathered}[/tex]

The graph of the hyperbola will be as following :

As shown in the figure :

vertices are : (-1,0) and (1,0)

Foci are ( -3.2 , 0) and (3.2 , 0)

End points are (0,-3) and (0,3)

Asymptotes are : y = 3x and y = -3x

The triangular faces of the prism shown are equilateral triangles with perimeter 30 cm. Use a net to find the surface area of the prism.

Answers

Explanation:

[tex]\begin{gathered} The\text{ surface area is made up of the two equilateral triangles shown above as well as the three rectangles.} \\ Area\text{ of Triangles = 2\lparen}\frac{1}{2}b*h) \\ If\text{ the perimeter of the triangle is 30cm, the length of one side = 30/3 = 10 = base} \\ Area\text{ = 2\lparen}\frac{1}{2}*10*8.7) \\ \text{ =87} \\ Area\text{ of the three rectangles = 3\lparen length*width\rparen } \\ \text{ =3\lparen10*12\rparen} \\ \text{ =360} \\ Total\text{ Surface Area = 360 + 87 = 447} \end{gathered}[/tex]

Surface Area of the two triangles in the net = 2*(0.5*b*h)

= 2*(0.5*10*8.7)

=87

Surface Area of three rectangles in the net = 3(l*b)

= 3*12*10

=360

Answer: Total Surface area = 360 + 87 = 447

Which of the following statements about the graph of f (x)=(0.5)^x shown above are true? Select all that apply.

Answers

From the graph, the following are true

[tex]1)Thefunctionf(x)=(0.5)^xis\text{ a decay function.}[/tex][tex]2)\text{The range of the function is the set of all real numbers }>0[/tex][tex]3)\text{ The graph of the function intersects the x-axis at x = 5}[/tex]

The population of a culture of bacteria, P(t), where t is time in days, is growing at a rate that is proportional to the population itself and the growth rate is 0.2. The initial population is 10.

Answers

Answer

Explanation

Using the formula for the population growth:

[tex]P(t)=P_0\cdot(1+r)^t[/tex]

where P₀ is the initial population, r is the rate of growth, and t is the time.

From the given information, we know that:

• P₀ = 10

,

• r = 0.2

1.

And we are asked to find P(50) (when t = 50), thus, by replacing the values we get:

[tex]P(50)=10\cdot(1+0.20)^{50}[/tex][tex]P(50)\approx91004.3815[/tex]

2.

For the population to double, this would mean that P(t) = 2P₀. By replacing this we get:

[tex]2P_0=10e^{0.20t}[/tex][tex]2(10)=10e^{0.20t}[/tex][tex]20=10e^{0.20t}[/tex][tex]\frac{20}{10}=e^{0.20t}[/tex][tex]\ln\frac{2}{1}=\ln e^{0.20t}[/tex][tex]\ln2=0.20t[/tex][tex]t=\frac{\ln2}{0.20}\approx3.5days[/tex]

Can you help me with this math question? it says "A cell phone plan costs $200 to start. Then there is a $50 charge each month, Write an expression that shows the total cost for x months on this plan" Is there a proportional relationship between time and cost of the cell phone plan?

Answers

Cost = 200 + 50x

The relationship between time and cost of the plan is not proportional. By definition, proportional ;relationships between two variables have equivalent ratio; one variable is always a constant value times the other which in this case is not .

To illustrate,

Month 1 Cost = 200 + 50(1) = 250 Ratio ( time:cost) = 1:250

Month2 Cost = 200 + 50(2) = 300 Ratio ( time:cost) = 2:300 = 1:150

Month 3 Cost = 200 + 50(3) = 350 Ratio ( time:cost) = 3:350

Month 4 Cost = 200 + 50(4)= 400 Ratio ( time:cost) = 4 :400 = 1:100

The ratios are not equivalent,thus the relationship is not proportional.

NO LINKS!! Use the method of to solve the system. (if there's no solution, enter no solution). Part 2z​

Answers

Answer:

smaller x-value:   (-4, 18)

larger x-value: (3, 11)

Step-by-step explanation:

Solving for x:

y = x^2 + 2

x + y = 14 ---> y = 14 - x

14 - x = x^2 + 2

0 = x^2 + x - 12

0 = (x + 4)(x - 3)

x = -4 or 3

Solving for y:

If x = -4

y = 14 + 4

y = 18

if x = 3

y = 14 - 3

y = 11

Answer:

[tex](x,y)=\left(\; \boxed{-4,18} \; \right)\quad \textsf{(smaller $x$-value)}[/tex]

[tex](x,y)=\left(\; \boxed{3,11} \; \right)\quad \textsf{(larger $x$-value)}[/tex]

Step-by-step explanation:

Given system of equations:

[tex]\begin{cases}\phantom{bbbb}y=x^2+2\\x+y=14\end{cases}[/tex]

To solve by the method of substitution, rearrange the second equation to make y the subject:

[tex]\implies y=14-x[/tex]

Substitute the found expression for y into the first equation and rearrange so that the equation equals zero:

[tex]\begin{aligned}y=14-x \implies 14-x&=x^2+2\\x^2+2&=14-x\\x^2+2+x&=14\\x^2+x-12&=0\end{aligned}[/tex]

Factor the quadratic:

[tex]\begin{aligned}x^2+x-12&=0\\x^2+4x-3x-12&=0\\x(x+4)-3(x+4)&=0\\(x-3)(x+4)&=0\end{aligned}[/tex]

Apply the zero-product property and solve for x:

[tex]\implies x-3=0 \implies x=3[/tex]

[tex]\implies x+4=0 \implies x=-4[/tex]

Substitute the found values of x into the second equation and solve for y:

[tex]\begin{aligned}x=3 \implies 3+y&=14\\y&=14-3\\y&=11\end{aligned}[/tex]

[tex]\begin{aligned}x=-4 \implies -4+y&=14\\y&=14+4\\y&=18\end{aligned}[/tex]

Therefore, the solutions are:

[tex](x,y)=\left(\; \boxed{-4,18} \; \right)\quad \textsf{(smaller $x$-value)}[/tex]

[tex](x,y)=\left(\; \boxed{3,11} \; \right)\quad \textsf{(larger $x$-value)}[/tex]

15. Deanna started a savings
account for her
when she was bom. She put
$1,500 in an account with a
simple 3.25% interest rate. What
will be the total amount in the
account after 18 years?
granddaughter

Answers

$23,775.00 will be the total amount in the account after 18 years, Formula of simple interest = A = P(1 + rt).

What is simple interest?

The straightforward interest formula makes figuring out how much interest will be applied to a loan quick and simple. Multiply the principle, this same number of days between payments, as well as the daily interest rate to determine simple interest.

Although this method of calculation is used in some mortgages, this type of interest is typically associated with auto loans as well as short-term loans.

Simple interest is calculated by multiplying the principle by the daily interest rate and number of days among payments.Simple interest rewards borrowers for making on-time or early monthly payments on their loans.Auto loans as well as short-term personal loans are two common uses for simple interest loans.

A represents the total amount of accrued interest and principal.
P is the principal amount.
The interest rate is I.
The annual percentage real interest rate, abbreviated as r, is 1/10.
R is the interest rate in annual percentage terms; R = r * 100 t is the time period in months or years.

In light of the query

P=$1500

r=3.25%

=0.0325

t=18 years

simple interest = 1500(1+ 0.0325 x 18)

=$(1500+877.5)

=$ 23,775

Thus the simple interest is  $23,775

Learn more about Simple interest from the link below
https://brainly.com/question/21020925
#SPJ1

Solve the following inequality. Write the solution set in interval notation

Answers

Given:

Inequality is

[tex]5(x-3)<2(3x-1)[/tex]

To find:

The solution set of the given inequality:

Explanation:

[tex]\begin{gathered} 5(x-3)<2(3x-1) \\ 5x-15<6x-2 \\ 5x-6x<15-2 \\ -x<13 \\ x>-13 \end{gathered}[/tex]

Therefore the solution set is

[tex](-13,\hat{\infty)}[/tex]

Final answer:

The solution set is

[tex](-13,\infty)[/tex]

Other Questions
look at the example problem. How would the partial products and the sum of the partial products change if there were 6 packages instead of 60? Explain. Answer this i have school tomorrow. stop letting me suffer answer please. please help asap please please ill give brainliest What is the domain of the cubic function f(x)= 9x + 2x-12 ? How many miles did a plane travel if it flew 455 miles per hour in 3 hours? Solve the system of linear equations using the graphing method. Use no solution and infinitely many when appropriate. List from greatest to least:5.1 , 5 1/5, 5.5 and 5 1/4 A smaller number and a larger number add up to 8 and have a difference of 6. (Let X be the larger number and Y be the smaller number) the paper didnt explain how to do this clearly could you take me step by step how to do this What kind of sentence is this?Why is your position on sales tax so different from your stance toward related issues?declarativeSubmitimperativeinterrogativeexclamatory The sequence below shows the number of trees that a nursery plants each year.2, 8, 32, 128 ...Let an represent the current term in the sequence and an-1 represent the previous term in thesequence. Which formula could be used to determine the number of trees the nursery will plantin year n? Find the value of each variable in the parallelogram.5u 10162u + 2V3U =0V= homework practice what is the 5th term of this sequence? According to Coulomb's law, the electrical force between two charged objects:A.is zero if they are opposite charges.B.increases with increasing charge.C.does not depend on the amount of charge.D.increases with increasing distance. X^3+3x^2+kx-10 and x-1 is a factor what is k Aircraft carriers use catapults to launch jets from their deck. One such catapult accelerates a 18,900 kg aircraft from rest to 61 m/s in 3.3 s. What is the magnitude of the force (in N) required to do this? Cada cuanto se produce un eclipse lunar In the given figure, mBJ = 106 and FHJH. Which statement is true?FHGFigure not drawn to scaleK106OA.The measure of ZG is 21", and triangle FGH is isosceles.OB.The measure of ZG is 56", and triangle FGH is isosceles.O.C.The measure of ZG is 21", and triangle FGH is not isosceles.The measure of ZG is 56", and triangle FGH is not isosceles.D. The Haber process for producing ammonia commercially is represented by the equation N2(g) + 3H2(g) 2NH3(g). If 7 L of NH3 are consumed, how many liters of H2 gas are required? i need to know how to solve this question please Can you help me please? If there is a solution whats the solution as well